Autor Nachricht
D2
BeitragVerfasst am: 20. Okt 2012 21:24    Titel:

Superpositionsprinzip ist doch eine tolle Sache,

die Messungen vor dem Doppelspalt zerstören die Interferenz,
nach dem Spalt anscheinend, kann man sich(was Messvorrichtungen betrifft) richtig austoben.
Und die Experimente ohne Kondensator funktionieren auch.

"Dieses Experiment hatte Feynman für technisch undurchführbar gehalten, da die
de-Broglie Wellenlänge fÜr Elektronen sehr klein ist. Beispielsweise liegt sie f¨ur
Elektronen der Energie E = 50keV bei λ = h/p ≈ 0.05°A. Der Spaltabstand für
das Doppelspaltexperiment müsste dann ebenfalls in dieser Größenordnung liegen
und damit viel kleiner als ein Atomdurchmesser sein. Feynman führte es deshalb als
reines Gedankenexperiment ein. Es wurde allerdings mittlerweile in modifizierter
Form tatsächlich von einigen Gruppen in Elektronenmikroskopen realisiert"
S 7-9 von 26
http://tu-dresden.de/die_tu_dresden/fakultaeten/fakultaet_mathematik_und_naturwissenschaften/fachrichtung_physik/isp/skm/lectures/EPI_Studentenanleitung.pdf

Mit einer Biprisma für Elektronen (wegen zu kleinem Streuwinkel)
S 3 von 14 hat man sowas schon gemacht
http://www.uni-tuebingen.de/Teilchenoptik/html/fprakt/pdf/AnleitungITEM.pdf

Wird es auch funktionieren wenn ein durchgehender Doppeltkondensator im Spiel ist ?

P.S. Der Schirm ist dann die mittlere, positiv geladene Kondensatorplatte.
Kann man die Streifen auf dieser Platte erwarten?

s S17 von 60 Elektronenbeugung an der Kante
http://www.wmi.badw.de/teaching/Lecturenotes/Physik3/Gross_Physik_III_Kap_12.pdf
TomS
BeitragVerfasst am: 20. Okt 2012 18:35    Titel:

D2 hat Folgendes geschrieben:
Mir war wichtig zu wissen, dass die Anwesenheit der Felder das Interferenzbild zwar beeinflussen, nicht aber zerstören kann.

Ws gibt keinen Grund, warum externe Felder das Intereferenzbild zerstören sollten.
D2
BeitragVerfasst am: 20. Okt 2012 17:20    Titel:

TomS hat Folgendes geschrieben:
Ein Beitrag nur mit zwei Zitaten von mir - was willst du damit sagen?

Wenn ich zitiere, bleibt bei mir oft das Programm hängen wenn ich auf "Vorschau" gehe, ich muss dann wieder neu einlogen und editieren.
TomS
BeitragVerfasst am: 20. Okt 2012 17:13    Titel:

Ein Beitrag nur mit zwei Zitaten von mir - was willst du damit sagen?
D2
BeitragVerfasst am: 20. Okt 2012 17:07    Titel:

TomS hat Folgendes geschrieben:
Hallo D2, in dem (aus dem PDF übernommenen) Versuch von Möllenstedt hat handelt es sich nicht um die Änderung der Interferenz durch einen dem Doppelspalt nachgeschalteten Kondensator, sondern um eine Modellierung eines Doppelspaltes durch eine kondensatorartige Anordnung.

Mir war wichtig zu wissen, dass die Anwesenheit der Felder das Interferenzbild zwar beeinflussen, nicht aber zerstören kann.

TomS hat Folgendes geschrieben:

Zur Erklärung: das Elektron wechselwirkt an keiner Stelle mit einem Magnetfeld, das Magnetfeld ist außerhalb der Spule identisch Null. Es ist jedoch so, dass zur Beschreibung dieser Feldkonfiguration ein von Null verschiedenens Vektorpoetntial angenommen werden muss. Letzteres ist klassisch nicht nachweisbar, allerdings eben über quantenmechanische Interferenzexperimente.

Hier gibt es eine interessante Erklärung.

"The magnetic Aharonov–Bohm effect is also closely related to Dirac's argument that the existence of a magnetic monopole can be accommodated by the existing magnetic source-free Maxwell's equations if both electric and magnetic charges are quantized..

Like the electromagnetic potential A the Dirac string is not gauge invariant (it moves around with fixed endpoints under a gauge transformation) and so is also not directly measurable"
http://www.microsofttranslator.com/BV.aspx?ref=IE8Activity&a=http%3A%2F%2Fen.wikipedia.org%2Fwiki%2FAharonov%25E2%2580%2593Bohm_effect
TomS
BeitragVerfasst am: 20. Okt 2012 16:29    Titel:

Hallo D2, in dem (aus dem PDF übernommenen) Versuch von Möllenstedt hat handelt es sich nicht um die Änderung der Interferenz durch einen dem Doppelspalt nachgeschalteten Kondensator, sondern um eine Modellierung eines Doppelspaltes durch eine kondensatorartige Anordnung.

jh8979 hat Recht, mein Beispiel des Aharoniv-Bohm-Effektes ist wohl zu weit von der ursprünglichen Fragestellung weg.

Zur Erklärung: das Elektron wechselwirkt an keiner Stelle mit einem Magnetfeld, das Magnetfeld ist außerhalb der Spule identisch Null. Es ist jedoch so, dass zur Beschreibung dieser Feldkonfiguration ein von Null verschiedenens Vektorpoetntial angenommen werden muss. Letzteres ist klassisch nicht nachweisbar, allerdings eben über quantenmechanische Interferenzexperimente.
D2
BeitragVerfasst am: 20. Okt 2012 16:08    Titel:

TomS hat Folgendes geschrieben:
Ich kenn kein derartiges Experiment für elektrische Felder, wohl aber für magnetische. Der einfachste (und zugleich komplizierteste ;-) Fall ist der des Aharonov-Bohm-Effektes.

Bild Typische Versuchsanordnung³
http://de.wikipedia.org/wiki/Aharonov-Bohm-Effekt

Wird das Effekt (ein/ausschalten des M Feldes)beobachtet wenn eine zusätzliche Abschirmung durch den 2. Zylinder statt finden soll(s. Bild unten)?
Ich nehme an, dass die Magnetlinien des M Feldes durch die Wände des 1. Zylinders geschlossen sind und die Elektronen durch lokale M Felder der Zylinderwand selbst unterschiedlich abgelehnt werden.
jh8979
BeitragVerfasst am: 20. Okt 2012 15:46    Titel:

TomS hat Folgendes geschrieben:
Ich kenn kein derartiges Experiment für elektrische Felder, wohl aber für magnetische. Der einfachste (und zugleich komplizierteste ;-) Fall ist der des Aharonov-Bohm-Effektes.

Ich finde dass ist noch eine Spur anders als in der Originalfrage, da es beim AB-Effekt ja darum geht, dass das Feld gleich Null ist am Ort der Teilchen, aber nicht das Potential.

Aber wo wir schonmal dabei sind: Es gibt auch das elektrische Analogon dazu. Schoen erklaehrt in diesem Artikel in Physics Today: The Aharonov–Bohm effects: Variations on a subtle theme (pdf-datei).
D2
BeitragVerfasst am: 20. Okt 2012 15:06    Titel:

TomS hat Folgendes geschrieben:
Ich kenn kein derartiges Experiment für elektrische Felder..

Im allgemeinen Fall muss man die Schrödingergleichung für den feldfreien Fall sowie den Fall mit nicht-verschwindendem Feld lösen und die Verschiebung bzw. Verzerrung des Intereferenzmusters berechnen. Im hier vorliegenden Fall würde man zunächst einlaufende Kugelwellen (aus den Spalten) betrachten und diese dann in einem (abschnittsweise) homogenen E-Feld propagieren lassen. Dazu benötigt man keine "Teilchenbahnen".

Ich habe etwas gefunden s 18 von 60
http://www.wmi.badw.de/teaching/Lecturenotes/Physik3/Gross_Physik_III_Kap_12.pdf
Ich brauche etwas Zeit um diesen Versuch so zu verstehen, dass ich mitdiskutieren kann.
TomS
BeitragVerfasst am: 20. Okt 2012 14:27    Titel:

Ich kenn kein derartiges Experiment für elektrische Felder, wohl aber für magnetische. Der einfachste (und zugleich komplizierteste ;-) Fall ist der des Aharonov-Bohm-Effektes

http://pauli.uni-muenster.de/tp/fileadmin/lehre/teilchen/ss10/AhaBohmEffekt.pdf
https://www.univie.ac.at/physikwiki/images/5/53/Der_Aharonov-Bohm-Effekt.pdf

Im allgemeinen Fall muss man die Schrödingergleichung für den feldfreien Fall sowie den Fall mit nicht-verschwindendem Feld lösen und die Verschiebung bzw. Verzerrung des Intereferenzmusters berechnen. Im hier vorliegenden Fall würde man zunächst einlaufende Kugelwellen (aus den Spalten) betrachten und diese dann in einem (abschnittsweise) homogenen E-Feld propagieren lassen. Dazu benötigt man keine "Teilchenbahnen".
Chillosaurus
BeitragVerfasst am: 20. Okt 2012 01:11    Titel:

D2 hat Folgendes geschrieben:
[...]
Wenn aber das Elektron im Kondensatorfeld abgelenkt werden kann obwohl dieses als eine Welle unterwegs sein soll(Beweis: Interferenz) dann muss der Kondensator als eine Messvorrichtung betrachtet werden, die das Interferenzbild zerstört. Wo liegt mein Denkfehler?

Du kannst den Kondensator als eine Art Messvorrichtung betrachten, musst aber den Schirm mit einbeziehn.
Damit bestimmst du zum einen die Position: die Elektronen können nicht außerhalb der Kondensatorplatten passieren, zum wird die Geschwindigkeit in eine Ablenkung auf den Schirm umkodiert.
Die Unschärfe ist relativ groß. Von daher sehe ich keinen Grund für einen vollständigen Kollaps der Wellenfunktion. Das Interferenzbild wäre zwar schwächer (am besten beschrieben durch genannte Geschwindigkeitsunterschiede, die zu unterschiedlichen Ablenkungen und somit zur Verschmierung führen, sollte dennoch vorhanden sein, wenn man lange genug aufzeichnet.
D2
BeitragVerfasst am: 19. Okt 2012 19:24    Titel:

Chillosaurus hat Folgendes geschrieben:
[Ich denke, dass das Interferenzbild dann aber in der Praxis immer noch schwächer ausfallen sollte, da die Elektronen, in Abhängigkeit ihrer Geschwindigkeit unterschiedlich abgelenkt werden.

Aber wenn ich richtig verstanden habe, verneint die Quantenmechanik die Geschwindigkeit des Elektrons nach dem Doppelspalt, da nur auf dem Schirm das Elektron sich als ein Teilchen sichtbar macht, sein Flugbahn(auch einzelne Elektronen können mit sich interferieren) bleibt verborgen.
„Damit können sich Teilchen, also Materie, in der Quantenmechanik wie Wellen verhalten und auch interferieren“
„Nur wenn eine Gewinnung der „Welcher-Weg“-Information nie erfolgte .. ergibt sich hinter dem Doppelspalt ein Interferenzbild“ http://de.wikipedia.org/wiki/Interferenz_(Physik)
Wenn aber das Elektron im Kondensatorfeld abgelenkt werden kann obwohl dieses als eine Welle unterwegs sein soll(Beweis: Interferenz) dann muss der Kondensator als eine Messvorrichtung betrachtet werden, die das Interferenzbild zerstört. Wo liegt mein Denkfehler?
Chillosaurus
BeitragVerfasst am: 19. Okt 2012 19:03    Titel:

D2 hat Folgendes geschrieben:
Habe ich richtig verstanden, dass je nach Spannung( an der Elektronenkanone/am Kondesators an durchgehenden Platten) muss der Schirm entsprechend geneigt werden(oben nach Re.,bzw. unten nach Li.), um die ursprüngliche Breite der Streifen durchgehen zu erhalten?

Ja, so müsste es gehen.
Ich denke, dass das Interferenzbild dann aber in der Praxis immer noch schwächer ausfallen sollte, da die Elektronen, in Abhängigkeit ihrer Geschwindigkeit unterschiedlich abgelenkt werden.
D2
BeitragVerfasst am: 19. Okt 2012 18:52    Titel:

Habe ich richtig verstanden, dass je nach Spannung( an der Elektronenkanone/am Kondesators an durchgehenden Platten) muss der Schirm entsprechend geneigt werden(oben nach Re.,bzw. unten nach Li.), um die ursprüngliche Breite der Streifen durchgehen zu erhalten?
Chillosaurus
BeitragVerfasst am: 19. Okt 2012 18:43    Titel:

Wegen anderer Auftreffwinkel wird sich auch die Streifenbreite ändern.
D2
BeitragVerfasst am: 19. Okt 2012 18:16    Titel: Elektroneninterferenz in einem Kondensator

Meine Frage:
Kann man das Interferenzbild der Elektronen nach dem Doppelspalt(elektrisch leitend um einen Faradayschen Käfig zu bilden) mit Hilfe von Feldern(z.B. im Feld eines Kondensators) beliebig verschieben?
Abb. unten, Elektronenkanone(nicht gezeichnet) befindet sich Li.,
die Interferenzstreifen erscheinen auf dem Schirm Re.

Meine Ideen:
Ich persönlich erwarte eine Verschiebung der Interferenzstreifen nach unten so lange das elektrische Feld des Kondensators homogen ist, wenn aber der Kondensator in mehrere unterteilt wird, dann kann so eine Veränderung zu der Zerstörung der Interferenz führen, oder?

Powered by phpBB © 2001, 2005 phpBB Group